Test Bank – ACP: Gastroenterology

Esophageal Disorders

1. A 55-year-old man with long-standing gastroesophageal reflux disease (GERD) is found to have Barrett esophagus on a routine upper GI endoscopy. Four-quadrant biopsies show no dysplasia. He takes proton pump inhibitor (PPI) therapy every day, and he reports that his heartburn is under reasonable control. His physical examination is unremarkable.

What would you recommend regarding the treatment of this patient’s Barrett esophagus?
A. Start an endoscopic surveillance program to look for dysplastic lesions
B. Increase the PPI dose to maximally suppress acid secretion
C. Refer for antireflux surgery to decrease the chances of progression to esophageal adenocarcinoma
D. Refer for esophagectomy

Key Concept/Objective: To understand the treatment of Barrett esophagus

Barrett esophagus is a sequela of chronic GERD in which the stratified squamous epithelium that normally lines the distal esophagus is replaced by abnormal columnar epithelium. The diagnosis of Barrett esophagus is established when the endoscopist sees columnar epithelium lining the distal esophagus. Regular endoscopic surveillance for esophageal cancer has been recommended in patients with Barrett esophagus. Esophageal biopsy specimens are taken during surveillance endoscopy primarily to identify dysplasia, a histologic diagnosis suggesting a premalignant lesion. For fit patients with identified high-grade dysplasia, three management options are available: esophagectomy, endoscopic ablative therapy, or intensive surveillance (withholding invasive therapy until the biopsies show adenocarcinoma). This patient has no active dysplasia, so invasive therapy is not indicated; he needs active surveillance. There is no evidence that increasing the doses of PPI helps with the dysplastic changes. Several studies have shown that antireflux surgery does not effect a permanent cure for GERD in the majority of patients (they still need to take PPI after the surgery), and surgery is no better than medication for preventing the peptic and neoplastic complications of GERD. (Answer: A—Start an endoscopic surveillance program to look for dysplastic lesions)

2. A 36-year-old woman comes to a walk-in clinic complaining of dysphagia. She has not seen a doctor in many years. She says her swallowing difficulty started 10 days ago. She has lost 30 lb in the past 6 months. She also complains of diarrhea with bloody stools and decreased visual acuity. She has had occasional fevers for the past 4 or 5 months. An enzyme-linked immunosorbent assay (ELISA) is positive for HIV; the patient’s CD4+ T cell count is 25 cells/mm3. Her physical examination reveals fever and cachexia. Her funduscopic examination shows evidence of retinitis. There is no thrush in the mouth cavity. The patient’s chest and abdominal examinations are unremarkable; her cardiovascular examination shows tachycardia; and her stools are heme positive. Endoscopy reveals one large (12 cm), shallow ulceration in the esophagus, surrounded by normal-appearing mucosa.

What is the most likely organism causing this patient’s esophagitis?
A. Epstein-Barr virus
B. Herpes simplex virus (HSV)
C. Candida esophagitis
D. Cytomegalovirus (CMV)

Key Concept/Objective: To identify the most common etiologic agents for infectious esophagitis

Most esophageal infections are caused by Candida, HSV, or CMV, either alone or in combination. These organisms rarely infect the esophagus of normal persons but often cause esophagitis in patients whose immune system has been compromised by AIDS, by advanced malignancy, or by organ transplantation and the subsequent administration of immunosuppressive drugs. Other conditions that can be associated with esophagitis are diabetes, alcoholism, corticosteroid therapy, scleroderma, achalasia, and esophageal strictures. Dysphagia and odynophagia are the presenting symptoms in the majority of patients. CMV esophagitis often is only one component of a generalized CMV infection, and some 20% to 40% of patients with CMV esophagitis have systemic symptoms. In contrast, candidal and HSV esophagitis usually are not associated with infection in other organs, and systemic symptoms caused by these pathogens are uncommon. Oral lesions, thrush, or ulcers are commonly found in patients with candidal or HSV esophagitis, but not in those with CMV esophagitis. Epstein-Barr virus has not been associated with esophagitis. CMV tends to cause discrete, shallow esophageal ulcerations that are very elongated (up to 15 cm in length) and surrounded by normal-appearing mucosa. Some patients with AIDS can develop large esophageal ulcers in which no pathogenic organism can be identified by culture or biopsy. These ulcers resemble CMV ulcers. They do not respond to antibiotics; steroids and thalidomide have been used with success. Tissue sampling is necessary to differentiate these giant CMV ulcerations from the giant idiopathic esophageal ulcerations that can be associated with HIV infection. In this patient, the presence of a large solitary esophageal ulcer, signs of colitis and retinitis (suggesting a disseminated CMV infection), and the absence of mouth lesions make CMV the most likely diagnosis. (Answer: D—Cytomegalovirus [CMV])

For more information, see Spechler SJ: 4 Gastroenterology: I Esophageal Disorders. ACP Medicine Online (www.acpmedicine.com). Dale DC, Federman DD, Eds. WebMD Inc., New York, March 2003

Peptic Ulcer Diseases

3. A 57-year-old white man presents with a 2-week history of gnawing epigastric pain that seems to be relieved with food and antacid. His medical history is significant for hypertension. His medications include hydrochlorothiazide and 81 mg enteric-coated aspirin. He does not smoke.

Which of the following statements regarding ulcerogenesis is true?
A. The prevalence of Helicobacter pylori infection is much higher in patients with peptic ulcer disease than in age-matched control subjects, and it is clear that the presence of H. pylori is sufficient to cause peptic ulcers
B. Epidemiologic studies suggest that nonsteroidal anti-inflammatory drugs (NSAIDs) vary in their propensity to cause ulcers; the risk of ulcer is dependent on dosage, but that risk is moderated by buffering and enteric coating
C. Gastrinoma causes 10% of all peptic ulcers
D. Idiopathic peptic ulcers account for up to 20% of gastric and duodenal ulcers in the United States

Key Concept/Objective: To understand the pathogenesis of peptic ulcer diseases

Up to 20% of gastric and duodenal ulcers in the United States occur in patients who have no evidence of H. pylori infection, who deny taking NSAIDs, and who have normal serum gastrin concentrations. These ulcers are referred to as idiopathic peptic ulcers. The prevalence of H. pylori infection of the stomach is much higher in patients with duodenal ulcers and, to a somewhat lesser extent, in patients with gastric ulcers than in age-matched control subjects. In addition, cure of H. pylori infection with antimicrobial therapy markedly reduces recurrences of duodenal and gastric ulcers. The etiologic mechanism linking H. pylori infection and ulcerogenesis is not yet absolutely established, for the following reasons: (1) voluntary ingestion of H. pylori led to gastric H. pylori infection and to gastritis but not to ulcers; (2) duodenal or gastric ulcers develop in only 10% to 20% of individuals with H. pylori gastritis, implying that only certain people with additional genetic, anatomic, physiologic, or environmental risk factors are predisposed to ulcers or that only certain H. pylori strains are ulcerogenic; (3) H. pylori induces diffuse inflammation in the stomach, yet the strongest link between H. pylori and peptic ulcer is with focal duodenal bulbar ulcer; and (4) gastric H. pylori infection is as common in women as in men, yet duodenal ulcer is two to three times less common in women. Currently, H. pylori can be considered the most important risk factor for duodenal and gastric ulcers, but it is clear that the mere presence of H. pylori in the stomach is not sufficient to cause peptic ulcers. Epidemiologic studies suggest that NSAIDs vary in their ability to cause ulcers, but this issue is complicated by the difficulty of comparing equipotent doses of NSAIDs. All prescription or overthe-counter NSAIDs should be considered ulcerogenic, with the risk of ulcer dependent on dosages and other patient-related factors, particularly advanced age and previous ulcer history. Even low doses of aspirin used for prophylaxis of cardiovascular disease (75 to 325 mg/day) are ulcerogenic in humans. Neither buffering of aspirin nor enteric coating appears to reduce the incidence of clinically detected ulcer formation. Gastrinoma causes less than 1% of all peptic ulcers. Peptic ulcers develop in 95% of patients with gastrinoma (Zollinger-Ellison syndrome); ulcers occur most commonly in the duodenal bulb but are also seen in the postbulbar duodenum, jejunum, lower esophagus, and stomach. Multiple ulcers are present in up to 25% of cases of Zollinger-Ellison syndrome. (Answer: D—Idiopathic peptic ulcers account for up to 20% of gastric and duodenal ulcers in the United States)

4. Two months ago, a 53-year-old white man was diagnosed by esophagogastroduodenoscopy (EGD) as having an uncomplicated duodenal ulcer. At that time, the patient tested positive on rapid urease testing and was appropriately treated with a clarithromycin-based regimen for H. pylori. He now returns with recurrent epigastric pain. He has no other medical conditions. He has been maintained on a proton pump inhibitor. He denies using NSAIDs. His vital signs and physical examination are unremarkable. His complete blood count, serum electrolyte levels, and serum calcium level are all within normal limits. He is referred for an upper GI series and is found to have a recurrent duodenal ulcer. The patient’s fasting gastrin level is 500 pg/ml (normal value, < 100 pg/ml).

For this patient, which of the following statements is true?
A. An upper GI series that is diagnostic of a bulbar duodenal ulcer will preclude endoscopy
B. Treatment failure with clarithromycin-based regimens occurs in approximately 30% of cases of H. pylori infection
C. A positive serum antibody test (sensitivity and specificity > 90%) would indicate persistent infection and require retreatment with metronidazole, tetracycline, and bismuth, as well as continuation of a proton pump inhibitor
D. Ulcers refractory to pharmacotherapy are seen in acid hypersecretory states; this patient’s fasting gastrin level is diagnostic of the ZollingerEllison syndrome

Key Concept/Objective: To understand the diagnostic modalities used in peptic ulcer disease

Despite having a lower sensitivity and specificity than endoscopy, an upper GI series using barium and air (double contrast) may be favored by primary care physicians and patients over referral for endoscopy for suspected uncomplicated ulcer. An upper GI series offers lower cost, wider availability, and fewer complications. However, for troublesome and undiagnosed dyspepsia, an upper GI series may be superfluous, because a normal result will often necessitate endoscopy (endoscopy is more sensitive than radiography) and because an upper GI series showing a gastric ulcer will also necessitate endoscopy and biopsy to exclude gastric malignancy. In many patients, only a finding of a duodenal bulbar ulcer on an upper GI series will preclude endoscopy.
Antimicrobial agents with activity against H. pylori include metronidazole, tetracycline, amoxicillin, and clarithromycin. A 2-week course of a three-drug regimen that includes a proton pump inhibitor, clarithromycin, and amoxicillin has a success rate approaching 90%. The major causes of treatment failure are poor compliance with the regimen and clarithromycin resistance; the latter occurs in around 10% of current strains and is increasing with increased macrolide use in the population. Breath testing is more useful than serology in diagnosing failure of eradication of H. pylori or reinfection in patients who were previously treated for H. pylori infection, because the serology will usually remain positive for several months even after successful treatment. A fasting serum gastrin concentration can be used to screen for an acid hypersecretory state resulting from Zollinger-Ellison syndrome. Antisecretory drugs (especially proton pump inhibitors) can also raise serum gastrin levels modestly (to 150 to 600 pg/ml). Definitive documentation of an acid hypersecretory state requires quantitative gastric acid measurement (gastric analysis). (Answer: A—An upper GI series that is diagnostic of a bulbar duodenal ulcer will preclude endoscopy)

5. A 54-year-old man with a history of COPD and tobacco abuse presents for evaluation of burning epigastric pain and melena. The epigastric pain has persisted for several weeks; the melena began several hours ago. His current medical regimen includes albuterol and ipratropium bromide nebulizers, longterm oral steroids, and theophylline. He also reports that he recently used an NSAID for joint pain. On physical examination, the patient’s heart rate is 115 beats/min and his blood pressure is 98/45 mm Hg. There is evidence of orthostasis. Abdominal examination does not demonstrate tenderness, rebound, or rigidity. A complete blood count is significant for a hematocrit of 39%; serum electrolytes are within normal limits. The patient is admitted for volume resuscitation. EGD is performed, and the patient is found to have a gastric ulcer with a visible vessel. The lesion is treated by injection of epinephrine.

For this patient, which of the following statements is true?
A. Corticosteroids not only are ulcerogenic but also impair healing of preexisting ulcers
B. The patient’s hemoglobin concentration makes a significant GI bleed unlikely
C. To exclude a diagnosis of ulcerated gastric cancer, gastric ulcers should be followed endoscopically until they are completely healed
D. Patients with H. pylori–related gastric ulcers should be managed with antibiotics and a 4-week regimen of a high-dose proton pump inhibitor

Key Concept/Objective: To understand the treatment of a bleeding gastric ulcer

Patients with gastric ulcers should be followed endoscopically until complete healing has been achieved so that an ulcerated gastric cancer is not missed. Corticosteroids, which block cyclooxygenase-2 (COX-2) but not COX-1, are not ulcerogenic when used alone, though they impair healing of preexisting ulcers. However, when corticosteroids are used in combination with NSAIDs, the risk of ulcer formation is much greater than when NSAIDs are used alone. In the first several hours after an episode of acute ulcer bleeding, the hemoglobin concentration will not completely reflect the severity of the blood loss until compensatory hemodilution occurs or until intravenous fluids such as isotonic saline are administered. Thus, the pulse rate and blood pressure in the supine and upright positions are better initial indicators of the extent of blood loss than are red cell counts. Gastric ulcer associated with H. pylori should be treated with antibiotics. Because they are larger than duodenal ulcers, gastric ulcers take longer to heal. Thus, after antibiotic administration, the patient should be treated with an acid antisecretory agent for an additional 4 to 8 weeks. (Answer: C—To exclude a diagnosis of ulcerated gastric cancer, gastric ulcers should be followed endoscopically until they are completely healed)

6. A 43-year-old woman presents to establish primary care. Her medical history is significant for an uncomplicated duodenal ulcer, which she experienced 18 months ago. At the time of diagnosis, she was treated with a clarithromycin-based regimen for H. pylori. She has since been asymptomatic.

For this patient, which of the following statements is true?
A. Given a success rate of only 90%, eradication of H. pylori should be confirmed after completion of a course of ulcer therapy
B. Reinfection with H. pylori is common in the United States
C. Patients who experience recurrence of ulcer symptoms during the first
2 years after therapy should be assessed by EGD, a urea breath test, or fecal antigen test
D. The sensitivity of the urea breath test is unaffected by use of a proton pump inhibitor

Key Concept/Objective: To understand the mechanism and diagnosis of treatment failure for eradication of H. pylori

After a patient has completed a course of ulcer therapy for an H. pylori–related uncomplicated duodenal ulcer, it is acceptable to follow the patient clinically without confirming eradication, because most compliant patients will be successfully cured of their H. pylori infection. Reinfection with H. pylori remains an uncommon event in the United States (approximately one reinfection per 100 patients a year). If symptoms of an H. pylori–related duodenal ulcer do not recur within 2 years after antimicrobial therapy, the patient is probably cured. Those in whom recurrent ulcer symptoms develop within 2 years after therapy should be assessed by endoscopy, a urea breath test, or fecal antigen test. Because proton pump inhibitors can suppress H. pylori without eradicating it, use of these drugs should be avoided for 2 weeks before the urea breath test is administered to minimize false negative results. (Answer: C—Patients who experience recurrence of ulcer symptoms during the first 2 years after therapy should be assessed by EGD, a urea breath test, or fecal antigen test)

For more information, see Feldman M: 4 Gastroenterology: II Peptic Ulcer Diseases. ACP Medicine Online (www.acpmedicine.com). Dale DC, Federman DD, Eds. WebMD Inc., New York, November 2004

Diarrheal Diseases

7. A 28-year-old man comes to your clinic complaining of diarrhea of 10 months’ duration. He is a graduate student and is currently writing a thesis. He has been sexually active with men in the past but not during the past 2 years. He denies experiencing weight loss or other constitutional symptoms. He notes no blood in the stool. He has no medical history. He explains that he has not sought attention before now because the problem is intermittent, and he notes that he sometimes experiences constipation rather than diarrhea. You include irritable bowel syndrome in your differential diagnosis.

Which of the following descriptions is characteristic of irritable bowel syndrome?
A. Painless diarrhea that occurs during the day or night
B. Abdominal pain with defecation and an altered bowel habit
C. Painless, chronic watery diarrhea of moderate severity
D. Diarrhea associated with postprandial flushing and a drop in blood pressure

Key Concept/Objective: To know the characteristic clinical presentation of irritable bowel syndrome

Patients with chronic diarrhea in whom no other etiology is established are commonly diagnosed with irritable bowel syndrome or functional diarrhea. Irritable bowel syndrome is characterized chiefly by abdominal pain that is associated with altered bowel function, including constipation, diarrhea, or alternating diarrhea and constipation. A diagnosis of functional diarrhea is made when patients do not have prominent abdominal pain and have no evidence of other specific causes of diarrhea. Obviously, these diagnoses are reliable only if a thorough evaluation has been done to exclude other causes of diarrhea. Nevertheless, there are certain clues to the diagnosis of irritable bowel syndrome or functional diarrhea that should be sought by the physician. Features that suggest a diagnosis of irritable bowel syndrome include a long history of diarrhea, dating back to adolescence or young adulthood; passage of mucus; and exacerbation of symptoms with stress. Historical points that argue against irritable bowel syndrome include recent onset of diarrhea, especially in older patients; nocturnal diarrhea; weight loss; blood in stools; voluminous stools (> 400 g/24 hr); blood tests indicating anemia, leukocytosis, or low serum albumin concentration; or a high erythrocyte sedimentation rate. (Answer: B—Abdominal pain with defecation and an altered bowel habit)

8. A 32-year-old woman presents as a walk-in patient in the emergency department. She complains of nausea and diarrhea that began early that evening. She reports that she ate a sandwich at a fast-food establishment for lunch, and she began experiencing symptoms several hours later. She denies seeing blood in the stool. She reports no similar experiences in the past; she has no recent travel history, nor has she had any contacts with sick persons. She was treated with a 3-day course of antibiotics for an upper urinary tract infection 2 months ago and is otherwise healthy.

Which organism is the most likely cause of this patient’s acute diarrheal illness?

A. Campylobacter jejuni
B. Salmonella enteritidis
C. Staphylococcus aureus
D. C. difficile

Key Concept/Objective: To understand that diarrhea caused by preformed toxin occurs within hours of exposure

Most acute diarrheas (i.e., those lasting < 4 weeks) are caused by infections and are self-limiting. Most are caused by viruses (e.g., adenovirus, Norwalk agent, rotavirus), but some are caused by bacteria (e.g., Campylobacter, Salmonella, Shigella, Escherichia coli) and others by protozoa (e.g., Giardia lamblia, Entamoeba histolytica). One mechanism for acute diarrhea is ingestion of a preformed toxin. Several species of bacteria, such as S. aureus, C. perfringens, and Bacillus cereus, can produce toxins that produce so-called food poisoning (i.e., vomiting and diarrhea) within 4 hours of ingestion. In such cases, the bacteria do not need to establish an intraluminal infection; ingestion of the toxin alone can produce the disease. Symptoms subside after the toxin is cleared, usually by the next day; evidence of toxicity (e.g., fever) is minimal. (Answer: C—Staphylococcus aureus)

For more information, see Schiller LR: 4 Gastroenterology: III Diarrheal Diseases. ACP Medicine Online (www.acpmedicine.com). Dale DC, Federman DD, Eds. WebMD Inc., New York, April 2003

Inflammatory Bowel Disease

9. A 26-year-old man presents with intermittent crampy abdominal pain, diarrhea without noticeable blood, and weight loss of 15 lb over 10 months. The bowel symptoms, including the diarrhea, wake him from sleep. On a few occasions, he has had fevers, nausea, and vomiting. The patient is an architect, and he describes his work as being stressful; he resumed smoking cigarettes a year ago. His older brother has had similar symptoms but has not yet been evaluated. On examination, the patient is a slender man with normal vital signs. He has an oral aphthous ulcer and poorly localized lower abdominal to midabdominal tenderness without peritoneal signs. Anal and rectal examinations are normal, and a stool guaiac test is negative. Stool leukocytes are present. The hematocrit is 34%. Results of examination with flexible sigmoidoscopy are normal.

Which of the following is the most likely diagnosis for this patient

A. Irritable bowel syndrome
B. Acute appendicitis
C. Crohn disease
D. Ulcerative colitis
E. Colon cancer

Key Concept/Objective: To be able to distinguish inflammatory bowel disease from other disorders, and to be able to distinguish between Crohn disease and ulcerative colitis

The diagnosis of inflammatory bowel disease is suggested by the fact that the patient’s symptoms developed over a number of months, that the patient has an oral aphthous ulcer, that fecal leukocytes are present, that the patient has experienced weight loss and has anemia, and by the possibility that the patient’s brother has a similar problem. The presence of nocturnal symptoms and fecal leukocytes eliminates irritable bowel syndrome. The long course makes acute appendicitis unlikely, though either irritable bowel syndrome or acute appendicitis can occur in patients with inflammatory bowel disease. The history is not suggestive of colon cancer, especially given this patient’s young age and in the absence of an inherited polyposis syndrome. The factors favoring a diagnosis of Crohn disease over that of ulcerative colitis at this stage in the evaluation include the association of smoking with the onset of symptoms. Crohn disease is strongly associated with smoking, but smoking decreases the risk of ulcerative colitis. In addition, the negative results on flexible sigmoidoscopy essentially eliminate ulcerative colitis from consideration. (Answer: C—Crohn disease)

10. A 33-year-old woman with Crohn disease presents with a flare of disease activity consisting of fever, right lower quadrant pain, weight loss of more than 10% of body weight, guaiac-positive diarrhea, and macrocytic anemia. Her disease is limited to the small intestine and terminal ileum. Her examination is significant for a temperature 100.2° F (37.9° C), active bowel sounds, and right lower quadrant tenderness.

Which of the following statements is true for this patient?
A. The anemia is probably caused by folate deficiency
B. Sulfasalazine is first-line therapy and will probably be sufficient to control her symptoms
C. An aminosalicylate will be required to control this flare
D. Corticosteroids will be necessary to control her symptoms
E. She should be hospitalized and given infliximab

Key Concept/Objective: To understand the treatment of inflammatory bowel disease

This patient has moderate to severe Crohn disease, as judged on the basis of her fever, weight loss, abdominal pain without obstruction, and ability to continue oral intake. Sulfasalazine is unlikely to deliver much anti-inflammatory activity to the small bowel because sulfasalazine is poorly hydrolyzed into its component sulfa and active salicylate moieties until it comes into contact with colonic bacteria. Aminosalicylate would be helpful, but for symptoms of this severity, a corticosteroid will be necessary. Infliximab, an anti–tumor necrosis factor monoclonal antibody, is an option for treatment of severe Crohn disease in patients who are not responsive to salicylates, antibiotics, or steroids. Unless the small bowel mucosal disease is very extensive, the macrocytic anemia is most likely caused by a deficiency of vitamin B12, which is absorbed in the terminal ileum. (Answer: D—Corticosteroids will be necessary to control her symptoms)

11. Two 28-year-old men with inflammatory colonic disease are seen in clinic; one has ulcerative colitis and the other has Crohn disease. Each is concerned about complications of his disease.

Which of the following is a correct assessment of these two patients?
A. Each may have arthritis in both HLA-B27–related and non–HLAB27–related distributions
B. Kidney stones can occur in each but are more common in patients who have ulcerative colitis
C. Sclerosing cholangitis in a spectrum from mild to severe can occur in ulcerative colitis but not in Crohn disease
D. Erythema nodosum and peripheral joint manifestations of colitis secondary to inflammatory bowel disease follow a course independent of the bowel disease and should be treated with NSAIDs
E. These two men have toxic megacolon, which is a complication unique to ulcerative colitis

Key Concept/Objective: To know the extraintestinal manifestations of inflammatory bowel disease

Inflammatory bowel disease is associated with peripheral joint arthritis and other conditions, such as erythema nodosum, that are not HLA-B27–associated and whose manifestations correlate with those of inflammatory bowel disease. NSAIDs worsen inflammatory bowel disease and can lead to bowel disease becoming refractory. Arthritis of the axial skeleton is HLA-B27–related and progresses independently of intestinal disease. Kidney stones are seen primarily in Crohn disease of the small intestine and are caused by increased oxalate absorption associated with malabsorption of intestinal fat and the binding of calcium to fatty acids. Cholangitis occurs in both ulcerative colitis and Crohn colitis. Toxic megacolon can occur in infectious colitis and Crohn disease as well as ulcerative colitis. (Answer: A—Each may have arthritis in both HLA-B27–related and non–HLA-B27–related distributions)

For more information, see Hanauer SB: 4 Gastroenterology: IV Inflammatory Bowel Disease. ACP Medicine Online (www.acpmedicine.com). Dale DC, Federman DD, Eds. WebMD Inc., New York, June 2001

Diseases of the Pancreas

12. A 64-year-old woman presents to the emergency department with abdominal pain, nausea, and vomiting of 3 days’ duration. The pain is epigastric and radiates to her back. She takes no medications. The physical examination reveals a patient with mild obesity; the patient’s temperature is 100.3° F (37.9° C), and epigastric tenderness is present. An abdominal CT scan with contrast shows pancreatitis with enhancement of the entire gland and a 5 cm collection of fluid next to the pancreas.

Which of the following is the most accurate diagnosis for this patient?
A. Interstitial pancreatitis with an acute fluid collection
B. Interstitial pancreatitis with an uncomplicated pseudocyst
C. Necrotizing pancreatitis with a peripancreatic phlegmon
D. Interstitial pancreatitis with a pancreatic abscess

Key Concept/Objective: To understand the diagnosis of acute pancreatitis

The definitions used to differentiate acute from chronic pancreatitis have changed recently, and more precise definitions were developed to describe the complications of acute pancreatitis. An acute fluid collection is defined as a collection of fluid occurring in or around the pancreas early in the course of acute pancreatitis. These collections are seen as areas of low attenuation without a visible capsule on CT. They are quite common in acute pancreatitis, occurring in 30% to 50% of cases. Many of these acute fluid collections resolve, but some may persist and develop a visible capsule, at which time they should be termed a pseudocyst. Pseudocysts are defined as collections of fluid surrounded by a fibrous capsule. It takes at least 4 to 6 weeks for an acute fluid collection to develop a capsule and become a pseudocyst. Pseudocysts may remain sterile or may become secondarily infected. Pancreatic necrosis is defined as an absence of enhancement of pancreatic parenchyma after the infusion of intravenous contrast on contrast-enhanced CT (CECT). Acute necrotizing pancreatitis is defined by the presence of necrosis on CECT; it is subclassified as either sterile necrosis or infected necrosis. Acute interstitial pancreatitis is defined by the absence of these CECT findings of necrosis. Finally, pancreatic abscess is defined as a circumscribed collection of pus containing little necrotic tissue. What was formerly called infected pseudocyst is now termed pancreatic abscess. The term phlegmon was abandoned, because no consensus could be reached as to its definition. (Answer: A—Interstitial pancreatitis with an acute fluid collection)

13. A 52-year-old man with a history of poorly controlled diabetes mellitus presents to the emergency department with severe abdominal pain of 36 hours’ duration. He had a similar episode 4 months ago. Physical examination is significant for tachycardia, diminished bowel sounds, epigastric tenderness, and a papular rash on his knees. Laboratory studies are significant for the following: leukocytes, 15,000 cells/mm3; blood glucose level, 450 mg/dl; amylase level, normal.

Which of the following is the most likely diagnosis for this patient?
A. Acute on chronic pancreatitis
B. Gallstone pancreatitis
C. Alcoholic pancreatitis
D. Pancreatitis secondary to hypertriglyceridemia

Key Concept/Objective: To be able to recognize hypertriglyceridemia as a cause of pancreatitis

Many factors have been implicated as causes of acute pancreatitis. Gallstones and alcohol abuse account for 70% to 80% of all cases of acute pancreatitis. Other etiologies include sphincter of Oddi dysfunction; benign and malignant strictures of the pancreatic duct; congenital anatomic abnormalities and genetic disorders; drugs; toxins; trauma; infections; and metabolic causes. Some cases are idiopathic. Metabolic causes of acute pancreatitis include hypertriglyceridemia and hypercalcemia. Serum triglycerides generally need to be in excess of 1,000 mg/dl to produce acute pancreatitis. This is most commonly seen in type V hyperlipoproteinemia and is usually associated with diabetes mellitus. Acute pancreatitis can itself raise triglyceride levels, but not to this degree. The diagnosis is usually confirmed with a combination of laboratory tests and imaging studies. Serum amylase measurement has long been the most widely used confirmatory laboratory test. At least 75% of all patients will have elevations in serum amylase at the time of initial evaluation. The serum amylase level may be normal in some patients with acute pancreatitis associated with alcohol use and in those with hyperlipidemic pancreatitis (marked elevations in the triglyceride level can interfere with the laboratory assay for amylase); the serum amylase level may be normal in patients with acute pancreatitis if the measurement is made several days after the onset of symptoms. Measurement of serum lipase is often used as an adjunct to or in place of serum amylase as a confirmatory test. The presence of a papular rash on this patient is consistent with eruptive xanthomas, supporting the diagnosis of pancreatitis secondary to hypertriglyceridemia. (Answer: D—Pancreatitis secondary to hypertriglyceridemia)

14. A 22-year-old man comes to your clinic for evaluation of chronic abdominal pain. The patient has been experiencing pain for 1 year. Initially, the pain was episodic, but lately it has become constant. It is felt in the epigastrium and radiates to the back. Sometimes the pain is accompanied by nausea and vomiting. The patient denies having diarrhea. Physical examination is unremarkable. An upper endoscopy and abdominal CT scan are unremarkable. You suspect chronic idiopathic pancreatitis.

Which of the following would be the most appropriate test to confirm the diagnosis?
A. Measurement of serum trypsinogen
B. Measurement of serum amylase and lipase
C Direct pancreatic function tests
D. Abdominal ultrasound

Key Concept/Objective: To understand the different tests for assessing pancreatic function

Diagnostic tests for chronic pancreatitis include those tests that detect functional abnormalities and those that detect abnormalities of pancreatic structure. Serum amylase or lipase levels may be elevated during acute exacerbations, but these elevations are usually modest and are neither routinely present nor diagnostic for chronic pancreatitis. A low serum trypsinogen level (< 20 ng/ml) is highly specific for chronic pancreatitis, but the trypsinogen level only drops to this level in advanced disease. The bentiromide test utilizes the measurement in urine of a metabolite that can only be produced by the action of pancreatic enzymes. The bentiromide test is no longer available in the United States. A 72-hour stool collection for fat is the gold standard to detect steatorrhea. Steatorrhea is only seen in far-advanced chronic pancreatitis. Fecal levels of elastase and chymotrypsin are reduced in more advanced cases of chronic pancreatitis. Direct pancreatic function tests involve placing a tube into the duodenum to collect pancreatic juice. This test directly measures pancreatic output of enzymes or bicarbonate after stimulation with a secretagogue. These tests are the most sensitive tests available and are able to detect chronic pancreatitis at an earlier stage than any other test. They are particularly useful in diagnosing those patients with small-duct chronic pancreatitis, in whom alternative diagnostic tests are likely to miss the diagnosis. Abdominal ultrasonography is most likely to detect advanced abnormalities of pancreatic structure and is diagnostic in only 60% of patients. (Answer: C—Direct pancreatic function tests)

15. A 38-year-old woman is being evaluated for abdominal pain, which has been present for 2 months. Her medical history and physical examination are unremarkable, and she takes no medications. An abdominal CT scan reveals a 5 cm cystic structure in the pancreas.

Which of the following is the most appropriate step to take next in the treatment of this patient?
A. Percutaneous drainage
B. Endoscopic drainage
C. Surgical resection
D. Follow-up and repeat imaging in 6 weeks

Key Concept/Objective: To be able to recognize mucinous cystic neoplasms

Besides the commonly seen pseudocysts, a number of other cystic lesions may occur in the pancreas, including true cysts and cystic neoplasms. Serous cystic neoplasms are benign, but mucin-producing cystic neoplasms may follow a more malignant course. Mucinous cystic neoplasms present as large cystic collections (cystadenomas and cystadenocarcinomas) and may be relatively asymptomatic. Most cystic neoplasms occur in middle-aged patients, particularly women. They are often mistaken for pseudocysts and inappropriately treated as such. These cystic neoplasms may follow an initially benign course, but when they undergo malignant degeneration, outcomes are as poor as in patients with standard adenocarcinoma. The presence of a cystic collection of the pancreas in a middle-aged (particularly female) patient without a previous history of pancreatitis should immediately suggest a cystic neoplasm, not a pseudocyst. The diagnosis of a cystic neoplasm requires histologic evidence of epithelial or neoplastic tissue in the cyst wall. When these collections are mistaken for pseudocysts, treatment involves drainage, and no tissue is obtained to allow differentiation of a cystic neoplasm from a pseudocyst. The therapy of choice for cystic neoplasms is surgical resection, not drainage. (Answer: C—Surgical resection)

16. A 62-year-old woman presents to the emergency department complaining of intense abdominal pain, nausea, and vomiting for the past 48 hours. On physical examination, the patient is visibly uncomfortable, with a low-grade fever, mild tachycardia, and normal blood pressure; her upper abdomen is markedly tender. Laboratory tests are remarkable for an amylase of 1150, bilirubin of 2.5, and creatinine of 2.3.

Which of the following is the most useful imaging test to determine whether this patient’s pancreatitis is caused by gallstones?
A. Plain film
B. Ultrasonography
C. CT scan
D. Endoscopic retrograde cholangiopancreatography (ERCP)

Key Concept/Objective: To understand the role of abdominal ultrasonography in acute pancreatitis

Ultrasonography is recommended in the initial evaluation of all pancreatitis to rule out obstruction caused by gallstones. It is more sensitive than CT for the diagnosis of gallstone disease, though CT is usually better at demonstrating morphologic changes in the pancreas caused by inflammation. Findings on plain film (such as the colon cutoff sign; enhancement of perirenal fat caused by retroperitoneal inflammation that creates a halo around the left kidney; or an abnormal duodenal loop) can suggest the diagnosis of pancreatitis but do not reveal its cause. ERCP does not play a role in the diagnosis of pancreatitis but can be useful in its management. (Answer: B—Ultrasonography)

17. A 50-year-old man comes to your clinic complaining of intermittent upper abdominal pain that radiates to his back and worsens with meals. He has a long history of binge drinking. He notes that lately he has been losing weight and that his stools have been loose.

Which of the following should be the first test to determine whether this patient has chronic pancreatitis?
A. Plain film
B. Ultrasonography
C. CT of the abdomen
D. ERCP
E. Secretin test

Key Concept/Objective: To know the stepwise approach to the diagnosis of chronic pancreatitis

The diagnosis of chronic pancreatitis can be made with an appropriate clinical history and demonstration of calcification of the pancreas on plain film. This should therefore be the first imaging test, though at most only 30% of patients with chronic pancreatitis will have this finding. If plain films are unrevealing, ultrasonography may demonstrate the characteristic findings of focal or diffuse enlargement of the pancreas, ductal irregularity and dilatation, and fluid collections adjacent to the gland. CT has a higher sensitivity than ultrasound (> 90%) and is the next step in diagnostic imaging. ERCP is the gold standard for diagnosing chronic pancreatitis on the basis of ductal abnormalities; the degree of ductal abnormalities correlates roughly with exocrine dysfunction. The secretin test, in which duodenal contents are sampled before and after secretin is administered intravenously, is probably the most sensitive direct assessment of pancreatic exocrine function, but because of improvement in imaging tests, the secretin test is used infrequently. (Answer: A—Plain film)

For more information, see Forsmark CE: 4 Gastroenterology: V Diseases of the Pancreas. ACP Medicine Online (www.acpmedicine.com). Dale DC, Federman DD, Eds. WebMD Inc., New York, October 2003

Gallstones and Biliary Tract Disease

18. A 15-year-old girl presents to the emergency department complaining of right upper quadrant pain. She has had two similar episodes in the past 3 months. The pain started 3 hours ago and increased rapidly. The pain is located in the epigastrium and radiates to the right shoulder. The patient is also complaining of nausea and vomiting. At presentation, she says the pain is starting to disappear. On physical examination, the patient has tenderness to palpation in the right upper quadrant. Laboratory testing shows a white cell count of 7,000, a hematocrit of 26%, and a normal platelet count. Her liver function test results are significant only for an indirect bilirubin of 2 mg/dl. Ultrasonography shows three stones in the gallbladder, no pericholecystic fluid, and no gallbladder wall edema. The common bile duct measures 4 mm.

Which of the following statements is the most accurate regarding this patient?
A. The patient should be started on antibiotic therapy; in 2 to 3 days, after this acute process resolves, a cholecystectomy should be performed
B. The patient has acute viral hepatitis; the gallstones are an incidental finding
C. If the patient undergoes a cholecystectomy, an analysis of the gallstones is likely to show black pigment stones
D. An endoscopic retrograde cholangiopancreatography (ERCP) should be done, because it is likely that a stone has passed to the common bile duct and is now causing obstruction

Key Concept/Objective: To understand the processes that lead to the formation of gallstones

Two principal types of stone, the cholesterol stone and the pigment stone, form in the gallbladder and biliary tract. The cholesterol stone is composed mainly of cholesterol (> 50% of the stone) and comprises multiple layers of cholesterol crystals and mucin glycoproteins. Mixed gallstones contain 20% to 50% cholesterol. The pigment stones contain a variety of organic and inorganic components, including calcium bilirubinate (40% to 50% of dry weight). Black pigment stones are most often seen in patients with cirrhosis or hemolytic anemia and are found predominantly in the gallbladder. This patient likely has biliary colic secondary to gallstones. Her laboratory results show evidence of hemolysis (low hematocrit, increased indirect bilirubin). Acute cholecystitis is unlikely in this clinical scenario because the pain is starting to disappear after 3 hours, there is no fever, and there is no evidence of leukocytosis on complete blood count. Also, ultrasonography did not show evidence of acute cholecystitis, such as the presence of pericholecystic fluid or edema of the gallbladder. Acute viral hepatitis can present as right upper quadrant pain; however, it is unlikely in this case because the pain is acute and is starting to resolve, and the only abnormal liver function test result is the indirect bilirubin value, suggesting hemolysis. An ERCP is not indicated because there is no evidence of obstruction or cholestasis, such as an elevation in the direct bilirubin level or the alkaline phosphatase level or a finding of a dilated common bile duct on ultrasound. (Answer: C—If the patient undergoes a cholecystectomy, an analysis of the gallstones is likely to show black pigment stones)

19. You are asked to consult regarding a 52-year-old man with fever who is in the surgical intensive care unit.
The patient has been in the hospital for 6 weeks after being injured in a car accident. He had a cranial fracture and multiple rib fractures; three feet of his jejunum were surgically removed, and he has had multiple complications since then, including pneumonia, sinusitis, and coagulase-negative Staphylococcus bacteremia. All of these complications seem to have resolved with adequate treatment. Over the past 2 days, he has developed increasing fever. He is still intubated and on total parenteral nutrition. On physical examination, the patient’s temperature is 102° F (38.9° C), his heart rate is 104 beats/min, and his blood pressure is 124/76 mm Hg. The patient has jaundice, and there is tenderness in the right upper quadrant. The examination is otherwise unchanged from previous notes in the chart. His complete blood count shows a white blood cell (WBC) count of 22,000 with left shift; the WBC count has been increasing over the past 2 days. Liver function testing shows a direct bilirubin level of 2.5 mg/dl and an alkaline phosphatase level of 415 mg/dl; these values were normal 3 days ago. An abdominal ultrasound done yesterday shows a thickened gallbladder wall with pericholecystic fluid and no sludge or stones. The chest x-ray is unchanged; blood cultures done yesterday are negative so far. The patient is receiving vancomycin for his Staphylococcus bacteremia. The primary team thinks that the ultrasound findings are not significant in this case, and they are looking for another source of fever.

Which of the following would be your recommendation?
A. Continue to follow cultures and wait for results before doing further workup
B. Recommend cholescintigraphy (HIDA scan) to evaluate for acute cholecystitis; add antibiotics to cover gram-negative and anaerobic organisms
C. Recommend that a repeat abdominal ultrasound be performed in 72 hours; continue current antibiotic regimen
D. Recommend performing a CT scan to look for other sources of infection

Key Concept/Objective: To understand the presentation of acute acalculous cholecystitis

Cholelithiasis is present in 90% to 95% of patients with acute cholecystitis, and most patients have had previous attacks of biliary colic. Acute cholecystitis may present as an acalculous cholecystitis in 5% to 10% of patients. It is predominantly noted in older men who are critically ill after major surgery, severe trauma, or extensive burn injury. This patient has fever, right upper quadrant pain, elevated bilirubin and alkaline phosphatase levels, and ultrasound findings suggestive of acute cholecystitis. Cholescintigraphy is the most accurate method of confirming the clinical diagnosis of acute cholecystitis (calculous or acalculous); this procedure involves the intravenous injection of technetium-99m– labeled hepatoiminodiacetic acid, which is selectively excreted into the biliary tree and enters the gallbladder. In the presence of cholecystitis, radiolabeled material enters the common bile duct but not the gallbladder. Because in this case the primary team does not think cholecystitis is an active problem, cholescintigraphy would be indicated to help confirm your presumptive diagnosis of acute acalculous cholecystitis; if confirmed, cholecystectomy would be recommended. Broad-spectrum antibiotic coverage is indicated in this patient, but there is still a need to find the etiology for his clinical deterioration. A CT scan would be appropriate only if the results of cholescintigraphy are negative. (Answer: B— Recommend cholescintigraphy [HIDA scan] to evaluate for acute cholecystitis; add antibiotics to cover gram-negative and anaerobic organisms)

20. A 54-year-old white man with a history of hypertension and diabetes presents to your clinic for followup after he was seen in a local emergency department with left flank pain. At that time, he had hematuria, and an ultrasound showed kidney stones. His liver function test results were normal. The report describes two stones in the left kidney. Also, as an incidental finding, three gallstones measuring 1 × 1 cm were seen; otherwise, the gallbladder was normal. The patient is concerned about the presence of these gallstones. On being asked about pain, he reports no episodes of pain except for the episode that caused him to visit the emergency department.

What would be your recommendation regarding the management of this patient’s gallstones?
A. Recommend not having surgery and continue to monitor clinically
B. Recommend cholecystectomy, because he has diabetes and he is at high risk for developing complications from acute cholecystitis in the future
C. Recommend surgery, because he is at high risk for developing gallbladder cancer in the next few years
D. Recommend oral ursodiol for dissolution of the stones

Key Concept/Objective: To know the appropriate treatment of asymptomatic cholelithiasis

Most gallstones are asymptomatic and are an incidental finding on ultrasonography performed for other reasons. Silent gallstones seldom lead to problems. Cholecystectomy is generally not indicated. Exceptions may be made for patients at increased risk for gallbladder cancer. In this case, the pain was related to nephrolithiasis (hematuria, left-sided pain, left kidney stones), and the patient has been otherwise asymptomatic. He has no risk factors for gallbladder cancer. Oral dissolution therapy is usually unsuccessful and requires long-term treatment. On the basis of this information, prophylactic cholecystectomy is not indicated for this patient; observation is the appropriate management. (Answer: A— Recommend not having surgery and continue to monitor clinically)

21. A 35-year-old man comes in for evaluation because his wife thinks he looks yellow. He feels fine, his medical history is unremarkable, and he takes no medications. On review of systems, he has no weight loss, anorexia, fevers, chills, or abdominal pains. He has no personal or family history of gallbladder problems. Lately, he has noted dark urine and pale stools. On examination, the patient’s vital signs are normal, but he is clearly jaundiced. His abdomen is nontender and free of organomegaly. Complete blood count and electrolyte and amylase levels are normal. An abdominal ultrasound shows multiple small gallstones in the gallbladder but none in the common bile duct. The common bile duct, however, is dilated.

Which of the following should be the next step in diagnosing this patient?
A. ERCP
B. Transhepatic cholangiography
C. CT
D. Repeat ultrasound
E. Cholescintigraphy

Key Concept/Objective: To understand the role of different imaging modalities in evaluating cholestatic jaundice and their ability to detect common bile duct stones

This patient has posthepatic cholestasis. Although ultrasound can often detect dilatation of the common bile duct, it may detect only 50% of common bile duct stones. CT and cholescintigraphy have similar limitations. In this case, ERCP is the procedure of choice because it will provide not only direct visualization of the common bile duct but also an opportunity to intervene therapeutically. If ERCP cannot be performed, transhepatic cholangiography is an alternative method for visualizing the bile ducts. (Answer: A—ERCP)

22. A 49-year-old man presents with right upper quadrant abdominal pain that began 8 hours ago. The pain is constant and is associated with nausea, vomiting, and fever. Over the past few months, he has had intermittent episodes of similar pain, but those were less intense, resolved spontaneously within 1 or 2 hours, and were never associated with vomiting or fever. Results of physical examination are as follows: temperature, 101.3° F (38.5° C); blood pressure, 130/90 mm Hg; pulse, 90 beats/min; and respirations, 16 breaths/min. The patient looks tired and moderately uncomfortable. Bowel sounds are present, but he has right upper quadrant tenderness. There is no palpable liver or gallbladder. Laboratory results are remarkable for a white blood cell count of 14,000, with a left shift. Bilirubin, amylase, and alkaline phosphatase levels are normal.

Which of the following is the best diagnostic imaging test for this patient?
A. Oral cholecystogram
B. HIDA scan
C. Ultrasound
D. CT scan
E. Plain abdominal x-ray

Key Concept/Objective: To understand the roles of various imaging modalities in the setting of acute cholecystitis

Ultrasound is the imaging test of choice. For detecting gallstones, it has a sensitivity of 88% to 90% and a specificity of 97% to 98%. It is noninvasive and readily available in most areas. If ultrasound results are equivocal, a HIDA scan can be performed to confirm the diagnosis of acute cholecystitis. HIDA scans are highly accurate, but they can be confounded by cirrhosis and can be misleading in patients who are fasting or who are receiving parenteral nutrition. CT is less sensitive than ultrasound. Oral cholecystograms are time-consuming. Because most gallstones are radiolucent, plain x-rays have limited usefulness. (Answer: C—Ultrasound)

23. The obese sister of the patient in Question 22 comes in the week after her brother’s visit with severe epigastric right upper quadrant pain that has been unrelenting for 24 hours. She has fevers, rigors, nausea, and vomiting. Results of physical examination are as follows: temperature, 102.2° F (39° C); blood pressure, 120/65 mm Hg; pulse, 100 beats/min; and respirations, 18 breaths/min. The patient looks ill and slightly jaundiced. She is shivering beneath a pile of blankets. Bowel sounds are hypoactive. There is marked right upper quadrant tenderness but no palpable liver or gallbladder. Laboratory results show a white blood cell count of 16,000 with a left shift. Bilirubin is 4.5, and alkaline phosphatase is 260. Her amylase level is normal.

Which of the following represents the diagnosis and best treatment for this patient?

A. Acute cholecystitis; treat with ampicillin-sulbactam
B. Acute cholecystitis; treat with ceftriaxone
C. Cholangitis; treat with ampicillin-sulbactam
D. Cholangitis; treat with ceftriaxone
E. None of the above

Key Concept/Objective: To be able to recognize the characteristic signs and symptoms of cholangitis and to select the appropriate antibiotic to cover likely organisms

This patient has the classic triad of jaundice, right upper quadrant pain, and fever with rigors (Charcot triad), which suggests cholangitis. If she also had shock and mental status changes (Reynold pentad), her prognosis would be grave: mortality in such patients approaches 50%. In addition to antibiotics and supportive care, patients who are very ill should be considered for biliary tract decompression (percutaneous or surgical decompression, or decompression with ERCP). The organisms that most commonly cause cholangitis are Escherichia coli, Klebsiella, enterococci, and Bacteroides fragilis. Ceftriaxone is not recommended in this case because it does not cover enterococci and has been associated with the development of gallbladder sludge. (Answer: C—Cholangitis; treat with ampicillin-sulbactam)

24. A 75-year-old man presents with gradually worsening pruritus, jaundice, and vague right upper quadrant abdominal ache. He has a 30-year history of ulcerative colitis. On exam, he has normal vital signs, scleral icterus, and hepatomegaly. His abdominal ultrasound shows dilated intrahepatic and extrahepatic ducts but no evidence of stones. His bilirubin level is 10, alkaline phosphatase level is 400, and amylase level is normal. An abdominal CT scan finds no pancreatic masses or adenopathy.

The differential diagnosis for this patient should include which of the following?

A. Primary biliary cirrhosis
B. Sclerosing cholangitis
C. Carcinoma of the biliary tract
D. Drug-induced cholestasis
E. B and C

Key Concept/Objective: To know that the differential diagnosis of cholestasis with ductal dilatation includes sclerosing cholangitis and ductal carcinoma

In this case, other possible diagnoses include a solitary common bile duct stone that escaped detection on ultrasound and CT, occult pancreatic carcinoma, bile duct stricture, and extrahepatic compression of the biliary tract. Although sclerosing cholangitis usually develops in younger men (aged 20 to 50 years), it is often associated with ulcerative colitis. About 60% of patients will also have a positive perinuclear antineutrophil cytoplasmic antibody (p-ANCA) test result. The hallmark finding on ERCP is segmental stenosis of the biliary tree. Primary biliary cirrhosis is an autoimmune disease that typically affects women. About 95% of patients have antimitochondrial antibodies. Both primary biliary cirrhosis and drug-induced cholestasis cause intrahepatic cholestasis without extrahepatic duct dilatation. (Answer: E—B and C)

For more information, see Ahmed A, Keeffe EB: 4 Gastroenterology: VI Gallstones and Biliary Tract Disease. ACP Medicine Online (www.acpmedicine.com). Dale DC, Federman DD, Eds. WebMD Inc., New York, December 2002

Gastrointestinal Bleeding

25. A 55-year-old man presents to the emergency department with a 2-day history of “coffee-ground emesis.” On physical examination, the patient is noted to have tachycardia and orthostatic hypotension. Volume resuscitation with intravenous normal saline is initiated; a complete blood count, coagulation studies, and routine chemistries are done, and esophagogastroduodenoscopy (EGD) is planned for further evaluation and management.

Which of the following is the most common cause of upper GI bleeding?
A. Mallory-Weiss tear
B. Variceal hemorrhage
C. Dieulafoy lesion
D. Peptic ulcer disease (PUD)

Key Concept/Objective: To understand that PUD is the most common cause of upper GI bleeding

Upper GI bleeding is arbitrarily defined as hemorrhage from a source proximal to the ligament of Treitz. Hematemesis essentially always reflects upper GI bleeding. Stools may range from black (melena) to bright red (hematochezia), depending on rates of bleeding and intestinal transit. The most common cause of upper GI bleeding is PUD, accounting for 60% of cases found on emergency endoscopy. About 50% of cases will have a cleanbased ulcer with a low probability of rebleeding, so that pharmacologic intervention is required. Adherent clots, visible vessels, or active bleeding portend less favorable outcomes unless endoscopic or surgical treatment is applied. Use of NSAIDs and Helicobacter pylori infection are the two most important risk factors; heavy alcohol ingestion and smoking are also associated with PUD bleeding risk. (Answer: D—Peptic ulcer disease [PUD])

26. A 39-year-old woman with a history of cirrhosis presents to the emergency department with massive hematemesis. Volume resuscitation with intravenous normal saline is initiated, and emergent EGD is planned for further evaluation and management. You are concerned about variceal bleeding.

Which of the following endoscopic interventions is considered first-line therapy in the management of esophageal varices?
A. Sclerotherapy
B. Band ligation
C. Thermal therapy
D. Injection therapy

Key Concept/Objective: To understand that band ligation is first-line therapy for the management of esophageal varices

With variceal bleeding, endoscopic treatment is used primarily for esophageal varices; the techniques include sclerotherapy and band ligation. Sclerotherapy utilizes a variety of sclerosants to induce variceal thrombosis, with sodium tetradecyl sulfate and ethanolamine oleate used most frequently. Complications include retrosternal chest pain, low-grade fever, ulceration (usually deep ulcers that heal within 3 weeks), dysphagia, delayed perforation (1 to 4 weeks later), and stricture formation. Complication rates vary from 19% to 35%. The popularity of sclerotherapy has diminished as a result of these complications. Intravariceal injections are more effective than paraesophageal injections in controlling bleeding. Band ligation is now considered the first-line endoscopic therapy for esophageal varices. The band ligator is readily attached to the distal end of the endoscope, which is advanced to the varix; the endoscopist then suctions the varix into the ligator cap and deploys a rubber band around the varix. This results in the plication of the varices and surrounding submucosal tissue, with fibrosis and eventual obliteration of varices. Comparative studies report better initial control of bleeding (91% versus 77%) and rebleeding rates (24% versus 47%) with band ligation than with sclerotherapy. Complications of banding include retrosternal chest pain, dysphagia from compromise of the esophageal lumen, band ulceration (usually superficial ulcers that heal within 2 weeks), overtube injury, and perforation. Complication rates vary from 2% to 19%. (Answer: B—Band ligation)

For more information, see Rajan E, Ahlquist DA: 4 Gastroenterology: X Gastrointestinal Bleeding. ACP Medicine Online (www.acpmedicine.com). Dale DC, Federman DD, Eds. WebMD Inc., New York, July 2003

Malabsorption and Maldigestion

27. A 36-year-old man presents to your clinic complaining of fatigue. His fatigue started 3 or 4 months ago.
His medical history is unremarkable. Review of systems is positive for occasional diarrhea, which the patient has been experiencing for several months, and for a 20-lb weight loss. Physical examination shows pallor. Occult blood is found on rectal examination. The rest of the examination is normal. Laboratory tests reveal iron deficiency anemia, and the patient tests positive on a qualitative fecal fat test. Results of an upper endoscopy and a colonoscopy are normal.

Which of the following tests would be most likely to provide helpful information in the workup of this patient?
A. Selenium-75–labeled homocholic acid-taurine (75SeHCAT) absorption test
B. Xylose absorption test
C. Bentiromide test
D. Tissue transglutaminase antibody

Key Concept/Objective: To understand that gluten-sensitive enteropathy (GSE) is a cause of iron deficiency anemia

Patients with GSE may present with a variety of complaints, including weight loss, fatigue, abdominal cramps, distention, bloating, and diarrhea. Other presentations include iron deficiency anemia, osteoporosis, and easy bruising. In a patient in whom the suspicion of GSE is high, a positive tissue transglutaminase antibody test makes the diagnosis almost certain. Alternatively, the diagnosis might rest on small bowel biopsy findings. Another helpful test is the identification of an endomysial antibody. The presence of fecal fat is helpful in this patient because it confirms the suspicion of an underlying malabsorptive disorder. The xylose absorption test evaluates the absorptive surface area of the small bowel. The bentiromide test is a noninvasive test to evaluate for pancreatic exocrine insufficiency. All of these tests can, however, be helpful in the evaluation of a patient with malabsorption and diarrhea. On the basis of this patient’s clinical presentation, the tissue transglutaminase antibody test is the one most likely to be helpful with the diagnosis. (Answer: D—Tissue transglutaminase antibody)

28. A 44-year-old woman with a history of GSE is evaluated for refractory disease. She was diagnosed with GSE 8 years ago. Her disease was initially well controlled with a gluten-free diet. Over the past few months, she has had persistent diarrhea and malabsorption that has not responded to her usual diet. Findings on physical examination are consistent with chronic malnutrition. An abdominal CT scan shows no masses or anatomic abnormalities that would account for her symptoms. An endoscopy is obtained, and small bowel biopsy shows villous atrophy and a layer of collagen underneath the enterocytes.

Which of the following is the most likely explanation for this patient’s symptoms?
A. Poor adherence to gluten-exclusion diet
B. Collagenous sprue
C. Small bowel lymphoma
D. Tropical sprue

Key Concept/Objective: To know that collagenous sprue is a possible complication of gluten-sensitive enteropathy

Collagenous sprue is a rare, devastating disease in which there is a layer of collagen underneath the enterocytes of the small bowel. The origin of collagenous colitis is unknown, but it develops in approximately half the patients who have refractory celiac disease. The symptoms are severe and include obvious malabsorption. The diagnosis is made on the basis of the classic histologic picture of villous atrophy and subepithelial collagen deposition. Therapy for collagenous sprue is uncertain; some patients respond to steroids. Poor adherence to gluten-exclusion diet is common; however, it would not explain the histologic changes seen in this patient. Small bowel lymphoma can be a complication of GSE; however, there is no evidence of this disorder on the imaging studies and biopsy. Tropical sprue is a malabsorptive disorder that appears in certain areas of the world. The diagnosis is based on the history of travel to endemic areas and a biopsy showing villous atrophy and inflammatory cells. (Answer: B—Collagenous sprue)

29. A 60-year-old man is being evaluated in your clinic for diarrhea. He started having diarrhea 6 months ago. He has undergone extensive evaluation over the past 2 months. His stool studies were consistent with steatorrhea. A stool culture for bacterial organisms was negative, as were stool studies for the presence of ova and parasites. An abdominal CT scan was normal. The patient has had arthritis for 5 years. Review of systems is positive for weight loss and occasional fever over the past 3 months. On physical examination, the patient’s temperature is 100.4° F (38° C). Skin hyperpigmentation and cervical lymphadenopathy are noted. You order an upper endoscopy. A small bowel biopsy shows villous atrophy and macrophages with sickleform particles. A periodic acid–Schiff (PAS) stain is positive.

Which of the following therapies is indicated for this patient?
A. Gluten-exclusion diet
B. Steroids
C. Trimethoprim-sulfamethoxazole (TMP-SMX)
D. Cholestyramine

Key Concept/Objective: To be able to recognize Whipple disease

The patient has clinical and laboratory findings consistent with the diagnosis of Whipple disease. This is a rare multisystem disease caused by infection with Tropheryma whippelii. Classically, the disease begins in a middle-aged man with a nondeforming arthritis that usually starts years before the onset of the intestinal symptoms. Arthralgias, diarrhea, abdominal pain, and weight loss are the cardinal manifestations of Whipple disease. Other complaints include fever, abdominal distention, lymphadenopathy, hyperpigmentation of the skin, and steatorrhea. The diagnosis rests on identifying the classic PAS-positive macrophages, which contain sickleform particles. The histologic lesion shows distended villi filled with the foamy, PAS-positive macrophages. Lymphatic dilatation is also present. A flat, villous surface can be seen in extreme cases. For treatment of Whipple disease, TMPSMX is given for 1 year. Gluten-exclusion diet is indicated in cases of GSE. Prednisone has no role in the treatment of Whipple disease. Cholestyramine is a resin that binds bile acids and can be used in cases of diarrhea related to bile acid malabsorption. (Answer: C— Trimethoprim-sulfamethoxazole [TMP-SMX])

30. A 10-year-old boy is evaluated for edema. He developed unilateral left upper and lower extremity edema 1 year ago. On review of systems, abdominal swelling and occasional diarrhea are noted. The physical examination is remarkable for unilateral edema and abdominal shifting dullness to percussion. A complete blood count shows lymphopenia. The serum albumin level is low at 1 g/dl. Urinalysis shows no protein; liver function tests are within normal limits.

Which of the following would be the most likely finding on small bowel biopsy for this patient?
A. Dilated lymphatics with club-shaped villi
B. Intense lymphocyte infiltration of the lamina propria
C. Eosinophilic invasion of the crypts on the small intestine
D. Lack of plasma cells

Key Concept/Objective: To be able to recognize intestinal lymphangiectasia

This patient has classic findings of congenital intestinal lymphangiectasia. Intestinal lymphangiectasia is often a congenital condition in which deformed lymphatics impair the transport of chylomicrons from the enterocytes to the mesenteric lymph duct. The blockage of lymphatic drainage may result in chylous ascites. Protein-losing enteropathy and lymphopenia are prominent features. Modest steatorrhea is also present. In the congenital form of the disease, lymphedema of the legs or of one leg and one arm is seen. With endoscopic examination, white villi, white nodules, and submucosal elevations may be noted. The white appearance of the mucosa is undoubtedly caused by retained chylomicron triacylglycerol. Double-contrast barium x-ray examination shows smooth nodular protrusions and thick mucosal folds without ulceration. On histologic examination, dilated lymphatics with club-shaped villi are seen. Lymphocytic infiltration of the lamina propria can be found in other disorders, such as lymphomas and immunoproliferative disease of the small intestine. Eosinophilic invasion of the crypts on the small intestine can be found in eosinophilic gastroenteritis. Lack of plasma cells can be found in patients with hypogammaglobulinemic sprue. (Answer: A—Dilated lymphatics with club-shaped villi)

31. A 32-year-old woman presents for evaluation of a sensitive tongue, which she has been experiencing for 2 weeks. She describes loss of appetite, weight loss of 15 lb over 3 months, and frequent (four or five times a day) loose stools. She denies having bloody stools, risk factors for HIV infection, or a personal or family history of gastrointestinal disease. She does recall briefly taking ciprofloxacin for traveler’s diarrhea while in Indonesia 18 months ago, which resolved with therapy. Review of systems is otherwise negative. Examination reveals normal conjunctiva; a swollen, tender tongue; a normal abdomen; and trace pedal edema. Laboratory tests show a hematocrit of 27, mild hypokalemia, and hypomagnesemia.

Which of the following pairs of interventions is most likely to help with this patient’s condition?
A. Folate and niacin
B. Iron sulfate and tetracycline
C. Gluten-free diet and prednisone
D. Folate and tetracycline
E. Azathioprine and prednisone

Key Concept/Objective: To be able to recognize the presentation and potential time course of tropical sprue and to understand the initial approach to therapy

Tropical sprue is a malabsorptive disease that occurs primarily in tropical locales among both residents and visitors. Its cause is unclear, but coliform bacteria have been selectively isolated from the jejunum of tropical sprue patients. The disease can occur while the patient is in the tropical locale, or it can present as late as 10 years after return. This patient spent time in Indonesia 18 months before presentation. Symptoms are generally much more apparent than in GSE and include prominent anorexia, weight loss, and diarrhea, as well as a high frequency of folate and vitamin B12 deficiencies with accompanying manifestations (macrocytic anemia, glossitis) and lower extremity edema. Folate should be administered as initial therapy; if the patient has had symptoms for a period longer than 4 months before the time of presentation, antibiotic therapy with tetracycline or a sulfonamide should be administered. Other than having glossitis, this patient has no evidence of niacin deficiency (pellagra). The patient’s anemia could be that of iron deficiency, but in light of the normal conjunctiva and other physical findings, that is less likely. A gluten-free diet and, in some cases, prednisone are required for treatment of GSE. Azathioprine and prednisone may be the initial regimen for a patient with Crohn disease. (Answer: D—Folate and tetracycline)

32. A 64-year-old man presents to his primary care physician with a complaint of foul-smelling diarrhea, which he has had for the past 4 to 5 months. He has three or four stools a day, which he describes as oily in nature. He denies experiencing a change in the caliber of his stools, and he also denies having abdominal pain, melena, or blood per rectum. His appetite is still fairly good, but he has lost 10 lb over the past 2 months and is somewhat fatigued. His medical history is notable for hypertension, hyperlipidemia, type 2 diabetes with retinopathy and mild neuropathy, and gastroesophageal reflux disease. His medications include metformin, insulin, atenolol, simvastatin, aspirin, and omeprazole. The neurologic examination is notable only for mild stocking-glove neuropathy, and an S4 is heard on cardiac examination. Laboratory tests reveal macrocytic anemia and mild hypoalbuminemia.

Which of the following is the most likely diagnosis for this patient?
A. Crohn disease
B. Intestinal lymphoma
C. Bacterial overgrowth syndrome
D. Hemochromatosis
E. Chronic pancreatitis

Key Concept/Objective: To understand the precipitants for the bacterial overgrowth syndrome

This patient has a subacute to chronic presentation with steatorrhea and likely folate deficiency, vitamin B12 deficiency, or both. He has diabetes mellitus, which can cause stasis through autonomic neuropathy, which is not uncommon in a patient with other diabetic complications. Anything that causes intestinal stasis allows a proliferation of bacteria, which leads to changes in bile salt metabolism and impaired absorption, primarily of vitamin B12. In addition, this patient is taking a proton pump inhibitor, which can both reduce motility and change the acid milieu of the proximal small bowel, often precipitating symptoms in a predisposed patient. Therapy usually entails repeated courses of antibiotics active against anaerobes. There is no convincing evidence for the effectiveness of any of the other choices presented. (Answer: C—Bacterial overgrowth syndrome)

33. A 38-year-old man with debilitating Crohn disease who is status post a 40 cm ileal resection presents for evaluation. He recounts progressive nonbloody diarrhea since his surgery 9 months ago, which is worse in the evening. He denies having abdominal pain, nausea or vomiting, fevers, chills, or sweats. He reports no recent travel, camping, or use of antibiotics. The exam is unrevealing. Chemistries show modest hypokalemia and a mild non–anion gap acidosis. Fecal fat quantitative analysis reveals minimal steatorrhea.

Which therapy is most likely to help this patient?
A. Cholestyramine
B. Loperamide
C. Tetracycline
D. High-protein, low-fat diet
E. Safflower oil before meals

Key Concept/Objective: To understand the effects of ileal resection on absorption, and recognize the appropriate therapy to minimize these effects

Ileal involvement is a common component of Crohn disease (regional enteritis) and may result in a poorly functioning ileum or even require resection. With moderate resections (30 to 100 cm), as in this case, malabsorption of bile salts is significant and results in bile salts entering the colon. This can lead to a secretory diarrhea known as choleretic enteropathy, the causal mechanism of which is bile salt-induced chloride secretion. Cholestyramine reduces the distal delivery of bile salts, thus lessening the diarrhea, and would be the most appropriate therapy in this patient. With larger ileal resections (> 100 cm), steatorrhea predominates, and cholestyramine may actually exacerbate the diarrhea. Therapy for patients undergoing larger resections is similar to that for patients with short bowel syndrome and includes antimotility agents such as loperamide and steps to increase the proportion of medium-chain fatty acids, which do not require bile salts for absorption. Safflower oil may also be used preprandially to act via peptide YY to slow gastric emptying, but it would be less useful in bile salt-induced diarrhea. Tetracycline is used to treat the bacterial overgrowth syndrome. (Answer: A—Cholestyramine)

For more information, see Mansbach CM III: 4 Gastroenterology: XI Diseases Producing Malabsorption and Maldigestion. ACP Medicine Online (www.acpmedicine.com). Dale DC, Federman DD, Eds. WebMD Inc., New York, September 2003

Diverticulosis, Diverticulitis, and Appendicitis

34. A 54-year-old man presents with a 3-day history of left lower quadrant pain. He reports that his appetite has decreased and that he has been experiencing mild nausea. He denies having any significant change in bowel function, hematochezia, or melena. On examination, his temperature is 101.3° F (38.5° C) and his blood pressure is 145/84 mm Hg. He has significant tenderness in the left lower quadrant, with some mild local fullness. No rebound is appreciated. The white blood cell count is 13,400.

What is the most appropriate diagnostic test for this patient?
A. Barium enema
B. Colonoscopy
C. Abdominal CT
D. Anoscopy
E. Diagnostic laparotomy

Key Concept/Objective: To understand appropriate diagnostic testing in presumptive diverticulitis

This patient presents with fairly severe clinical diverticulitis, and hospitalization is appropriate. Abdominal CT is the most useful test in this situation. It allows confirmation of the diagnosis as well as identification of complications. In this case, the possibility of abscess raised by the examination would be evaluated. Also, feasibility of percutaneous drainage would be determined. Colonoscopy is relatively contraindicated in cases of suspected diverticulitis because its use increases the risk of perforation. Barium enema is also not advisable, because its use increases the risk of peritoneal contamination if perforation is present. Water-soluble enema would be safer but would not provide the same degree of information as CT scanning. Most authorities would not recommend surgical intervention without initial diagnostic evaluation, because most patients do well without urgent surgery. Anoscopy would not be helpful; it is used primarily to evaluate very distal sources of GI bleeding. (Answer: C—Abdominal CT)

35. A 67-year-old woman presents to the office with 6 days of mild left-lower-quadrant cramping. She describes previous bouts of diverticulitis similar in character. She denies having fever, chills, nausea, or evidence of gastrointestinal bleeding. On examination, she is afebrile with mild left-lower-quadrant tenderness. A presumptive diagnosis of diverticulitis is made. She denies having any allergies to medication.

What is the most appropriate choice for antibiotic monotherapy?
A. Metronidazole
B. Amoxicillin-clavulanate
C. Cephalexin
D. Clindamycin
E. Azithromycin

Key Concept/Objective: To understand appropriate antimicrobial choice in diverticulitis

This patient presents with mild diverticulitis. It is important to use an antibiotic that provides good anaerobic and gram-negative coverage, making amoxicillin-clavulanate a good choice. Metronidazole and clindamycin provide good anaerobic coverage but insufficient coverage against gram-negative rods. These two drugs are frequently used in combination with trimethoprim-sulfamethoxazole or quinolones (to cover gram-negative rods). Azithromycin and cephalexin provide inadequate coverage for both anaerobes and gramnegative rods. (Answer: B—Amoxicillin-clavulanate)

36. A 52-year-old man presents for follow-up after undergoing colonoscopy because he has a history of colon cancer. He was reassured to learn that no polyps or tumors were seen but was told that he has diverticulosis. He asks what the chances are that this will cause future difficulties.

What is the likelihood that a patient with asymptomatic diverticulosis will go on to develop diverticulitis?
A. < 5%
B. 10% to 25%
C. 40% to 50%
D. 70% to 80%
E. > 90%

Key Concept/Objective: To understand the natural history of minimally symptomatic diverticulosis

Diverticulosis is common and is often discovered during evaluation for another problem (e.g., colon cancer). As more individuals undergo colon evaluation, more cases of asymptomatic diverticular disease will be identified. It is important to be able to educate patients on symptoms to watch for. It is also important to be able to inform them about the prognosis. Current data suggest that the likelihood of developing diverticulitis is 10% to 25%. An estimated 80% of patients with diverticulosis are completely asymptomatic. (Answer: B—10% to 25%)

37. Some months later, the patient in Question 36 comes to you for evaluation after experiencing two brief episodes of left-lower-quadrant cramping. He denies having any bleeding or severe symptoms. He has not required antibiotics or hospitalization. He is not particularly bothered by these symptoms but asks if there is anything he should be doing with respect to the diverticulosis.

Which of the following interventions would be most appropriate at this time?
A. Narcotic analgesic
B. Anticholinergic agent
C. Colonic resection
D. Fiber supplementation
E. Repeat colonoscopy

Key Concept/Objective: To understand the management of mild diverticulosis

In patients with asymptomatic or minimally symptomatic diverticulosis, an increase in the amount of fiber consumed—as would occur through eating more fruits and vegetables—appears to reduce the risk of symptoms and complications. Bran has been shown to improve constipation but not pain. There is no evidence to support the use of narcotic or anticholinergic agents. This patient underwent colonoscopy within the past year, so repeat colonscopy would be very unlikely to offer any additional information. (Answer: D—Fiber supplementation)

For more information, see Harford WV: 4 Gastroenterology: XII Diverticulosis, Diverticulitis, and Appendicitis. ACP Medicine Online (www.acpmedicine.com). Dale DC, Federman DD, Eds. WebMD Inc., New York, April 2001

Enteral and Parenteral Nutritional Support

38. A 74-year-old man is transported to the emergency department by ambulance for evaluation of cough, dyspnea, and altered mental status. Upon arrival, the patient is noted to be minimally responsive. Results of physical examination are as follows: temperature, 102.1° F (38.9° C); heart rate, 116 beats/min; blood pressure, 94/62 mm Hg; respiratory rate, 34 breaths/min; and O2 saturation, 72% on 100% O2 with a nonrebreather mask. The patient is intubated in the emergency department, and mechanical ventilation is initiated. Coarse rhonchi are noted bilaterally. A portable chest x-ray reveals good placement of the endotracheal tube and lobar consolidation of the right lower lobe. Laboratory data are obtained, including sputum Gram stain and culture and blood cultures. Empirical antimicrobial therapy is initiated, and the patient is admitted to the medical intensive care unit for further management. The intern on call inquires about the appropriateness of initiating nutritional support (enteral or parenteral feeds) at this time.

Which of the following statements regarding nutritional support is true?
A. Enteral nutrition is less likely to cause infection than parenteral nutrition
B. Parenteral nutrition has consistently been shown to result in a decrease in mortality, compared with standard care
C. The use of oral supplements in hospitalized elderly patients has been shown to be harmful
D. Parenteral nutrition is the preferred mode of nutrition in cancer patients because of its lower incidence of infections

Key Concept/Objective: To understand that enteral nutrition is less likely to cause infection than parenteral nutrition

Comparisons of enteral nutrition with parenteral nutrition have consistently shown fewer infectious complications with enteral nutrition. Elderly patients should receive supplemental feeding or enteral nutrition if they are incapable of eating adequately. A trial in 501 hospitalized elderly patients randomized to oral supplements or a ward diet showed that, irrespective of their initial nutritional status, the patients receiving oral supplements had lower mortality, better mobility, and a shorter hospital stay. The difference between ward diet and supplementation was even more pronounced in a secondary analysis of patients with weight loss. (Answer: A—Enteral nutrition is less likely to cause infection than parenteral nutrition)

39. A 32-year-old man with AIDS who is experiencing chronic diarrhea, anorexia, and wasting is referred for evaluation for nutritional support. Results of physical examination are as follows: temperature, 97.6° F (36.4° C); heart rate, 67 beats/min; blood pressure, 102/62 mm Hg; respiratory rate, 12 breaths/min; height, 70 in; and weight, 50 kg. The patient appears chronically ill; there is bitemporal wasting, and his hair is easily pluckable. The patient says he has friends with AIDS who are receiving “I.V. nutrition,” and he would like to know if such therapy would benefit him.

Which of the following statements regarding home total parenteral nutrition (TPN) is true?
A. Evidence demonstrates improved survival and quality of life in patients with metastatic cancer who are receiving home TPN
B. Evidence demonstrates improved survival and quality of life in patients with AIDS who are receiving home TPN
C. Evidence demonstrates improved survival and quality of life in patients with short bowel from Crohn disease who are receiving home TPN
D. No evidence supports the use of home TPN in any patient population

Key Concept/Objective: To understand which patients clearly benefit from home TPN

Patients with intestinal failure from a short bowel, chronic bowel obstruction, radiation enteritis, or untreatable malabsorption can be nourished by TPN at home. Long-term success has been achieved with a tunneled silicone rubber catheter or an implanted reservoir. Premixed nutrients are infused overnight. The catheter is then disconnected and a heparin lock applied, leaving the patient free to attend to daily activities. Survival of patients with short bowel resulting from the treatment of Crohn disease or pseudo-obstruction is excellent. Home TPN increases quality-adjusted years of life in these patients and is cost-effective. On the other hand, mean survival in AIDS patients or those with metastatic cancer who receive home TPN is about 3 months. There is no evidence that home TPN prolongs survival for these patients or enhances their quality of life. Trials are urgently required to justify the use of home TPN in patients with terminal cancer and AIDS. (Answer: C—Evidence demonstrates improved survival and quality of life in patients with short bowel from Crohn disease who are receiving home TPN)

For more information, see Jeejeebhoy KN: 4 Gastroenterology: XIII Enteral and Parenteral Nutritonal Support. ACP Medicine Online (www.acpmedicine.com). Dale DC, Federman DD, Eds. WebMD Inc., New York, January 2003

Gastrointestinal Motility Disorders

40. A 45-year-old man presents to your office complaining of nausea, early satiety, anorexia, and abdominal discomfort. His medical history is remarkable for a Roux-en-Y partial gastrectomy a few months ago. You suspect that the surgery has resulted in disruption of gastric motility.

Which of the following is most likely to relieve this patient’s symptoms?
A. Metoclopramide, 10 to 20 mg up to four times a day
B. Erythromycin lactobionate, 3 to 6 mg/kg every 8 hours
C. Further resection of the gastric remnant
D. Omeprazole, 20 mg two times a day

Key Concept/Objective: To know the most effective treatment for a patient with upper gut stasis resulting from gastric surgery

This patient most likely has upper gut stasis caused by his gastric surgery. This is most commonly the result of uncoordinated phasic pressure waves in the Roux limb. The vagotomized gastric remnant may also contribute to the development of symptoms because of derangements in its relaxation and contraction. In general, pharmacologic agents are ineffective in relieving symptoms in these patients. Further resection of the gastric remnant gives symptomatic relief in approximately two thirds of patients. (Answer: C—Further resection of the gastric remnant)

41. A 25-year-old woman presents to your clinic complaining of lower abdominal pain and periods of constipation alternating with episodes of diarrhea. Her previous physician diagnosed her with an irritable bowel after an extensive evaluation. She takes a selective serotonin reuptake inhibitor for depression but has no other significant medical history.

Which of the following abnormalities is NOT present in patients with functional gastrointestinal disorders?
A. Histologic changes, such as loss of normal villi, can be seen in small bowel biopsy
B. Abnormal gastrointestinal motility
C. Heightened visceral sensation
D. Psychosocial disturbance

Key Concept/Objective: To be able to identify the common pathogenetic features of the functional gastrointestinal disorders

Functional gastrointestinal disorders are characterized by disturbances in motor or sensory function in the absence of any known mucosal, structural, biochemical, or metabolic abnormality. These disorders include irritable bowel syndrome, functional dysphagia, nonulcer dyspepsia, slow-transit constipation, and outlet obstruction to defecation. The shared common pathogenetic features of these disorders are abnormal motility, heightened visceral sensation, and psychosocial disturbance. The loss of villi, as demonstrated on small bowel biopsy, is evidence of a mucosal abnormality and should prompt consideration of another diagnosis, such as celiac sprue. (Answer: A—Histologic changes, such as loss of normal villi, can be seen in small bowel biopsy)

42. For the past several months, a 50-year-old man has been experiencing upper abdominal discomfort, nausea, and bloating; these symptoms are worse exclusively after eating. You diagnose the patient with dyspepsia.

Which of the following is NOT one of the alarm features associated with dyspepsia caused by ulcers or cancer?
A. Dysphagia
B. Emesis
C. Weight loss
D. Bleeding

Key Concept/Objective: To know the clinical findings associated with dyspepsia caused by ulcers or cancer

Dyspepsia refers to symptoms of nausea, vomiting, upper abdominal discomfort, bloating, anorexia, and early satiety that usually occurs in the postprandial period. When such symptoms occur in the absence of a gastric or duodenal ulcer, the condition is referred to as nonulcer dyspepsia. This condition affects approximately 20% of the population of the United States. Certain clinical features suggest that dyspepsia may be associated with mucosal diseases, such as ulcers or cancer. The presence of these features makes exclusion of these disorders imperative. These alarm features include dysphagia, bleeding, and weight loss. Emesis is a nonspecific symptom frequently present in patients with dyspepsia. It should be noted, however, that cancer may be present in patients with dyspepsia despite the absence of these symptoms. (Answer: B—Emesis)

43. A 64-year-old man with a long history of poorly controlled diabetes is diagnosed as having gastroparesis on the basis of his medical history and transit tests showing delayed gastric emptying. He is referred to you for long-term treatment.

Which of the following should NOT be included in your treatment strategy?
A. Correction of dehydration and electrolyte and nutritional deficiencies
B. Prokinetic therapy with metoclopramide or erythromycin
C. Vagotomy
D. Antiemetics as needed

Key Concept/Objective: To know the major modalities used in the treatment of gastroparesis

The major treatment approaches for the patient with a gastric or small bowel motility disorder include correction of fluid, electrolyte, and nutritional deficiencies; the use of prokinetic agents such as metoclopramide and erythromycin; the use of antiemetic agents for symptomatic relief; suppression of bacterial overgrowth (if present); decompression in severe cases; and surgical resection if the disorder is determined to be isolated to one discrete area of the gut. Vagotomy can actually cause gastroparesis and should be avoided. (Answer: C—Vagotomy)

For more information, see Camilleri M: 4 Gastroenterology: XIV Gastrointestinal Motility Disorders. ACP Medicine Online (www.acpmedicine.com). Dale DC, Federman DD, Eds. WebMD Inc., New York, October 2002

Liver and Pancreas Transplantation

44. A patient with known chronic hepatitis C and cirrhosis comes to your office wondering about liver transplantation. He read on the Internet that the waiting time for liver transplantation is long and that one should be included “early” to improve the chances of getting a timely transplantation. The patient stopped smoking over 20 years ago and does not drink alcohol. He is otherwise in good health.

Which of the following is NOT a contraindication for orthotopic liver transplantation?
A. Ongoing abuse of drugs or alcohol
B. Significant cardiovascular or neurologic disease
C. Infection with hepatitis B or C
D. Extrahepatic malignancy
E. Extensive portal and splanchnic venous thrombosis

Key Concept/Objective: To know the contraindications for liver transplantation

Contraindications for liver transplanatation can be categorized into issues of abuse, underlying significant medical problems, malignancy, and technical limitations. Ongoing abuse of drugs or alcohol is one of the most frequent contraindications to transplantation. Patients with significant cardiovascular or neurologic disease cannot withstand the stress of transplantation. Although transplantation is often performed for hepatocellular carcinoma, extrahepatic malignancy is a contraindication. Extensive portal and splanchnic venous thrombosis prevents viable blood flow to the transplanted liver. Although patients with hepatitis B or C may require antiviral therapy before and after transplantation, having hepatitis B or C is not a contraindication. (Answer: C—Infection with hepatitis B or C)

45. A 40-year-old man with cirrhosis secondary to hepatitis B is being evaluated for orthotopic liver transplantation. He asks you what he should expect after his operation.

Which of the following statements is true regarding this patient?
A. There is a 1% to 2% chance of failure of his transplanted organ in the immediate postoperative period
B. Although he may develop progressive jaundice over time, suggesting ductopenic rejection, this has little consequence
C. His hepatitis B will most likely be cured after transplantation, so he will not need his antiviral medications anymore
D. Infections are among the most serious complications after liver transplantation
E. Thirty percent of patients taking cyclosporine or tacrolimus develop kidney failure within 10 years after transplantation

Key Concept/Objective: To know the most common and serious complications after liver transplantation

From 5% to 10% of liver transplant patients experience immediate graft failure. Although ductopenic rejection is a more indolent process, it usually creates a need for retransplantation. Hepatitis B is not cured by transplantation and can even cause rapidly progressive liver disease after transplantation; however, aggressive antiviral therapy before and after transplantation has been associated with prolonged graft longevity. Approximately 10% of patients treated with the calcineurin inhibitors cyclosporine or tacrolimus develop renal failure after transplantation. Because of the strong immunosuppressive agents required, infections remain among the most serious complications, both short-term and long-term, after transplantation. (Answer: D—Infections are among the most serious complications after liver transplantation)

46. A 37-year-old woman with a history of cryptogenic cirrhosis who underwent orthotopic liver transplanation 1 year ago asks you to assume her posttransplantation care.

Which of the following is true regarding this patient?
A. She can expect to return to work, but it is unlikely that she will be able to tolerate vigorous activity
B. The phenytoin she takes for her seizure disorder may result in an elevation in serum cyclosporine level, leading to a need for lower doses of cyclosporine
C. Infection is the leading cause of death in the posttransplantation population
D. Most transplant centers report 75% to 80% 5-year survival rates
E. If she develops hyperlipidemia as a result of taking cyclosporine, she is unlikely to benefit from a change in immunosuppressive medications

Key Concept/Objective: To understand the long-term prognosis of liver transplant recipients, in terms of both mortality and functional status

Many posttransplantation patients not only return to work but are able to participate in such vigorous activities as marathon running. Phenytoin induces the cytochrome P-450 system, leading to decreased serum levels of cyclosporine. Age-related cardiovascular disease is the leading cause of death in posttransplantation patients. Many patients receiving cyclosporine develop hyperlipidemia; some can be helped by changing this medication to tacrolimus. Because of advances in immunosuppressive medications and surgical techniques, most transplant centers report 5-year survival rates of 75% to 80%. (Answer: D—Most transplant centers report 75% to 80% 5-year survival rates)

47. A 28-year-old patient with type 1 diabetes mellitus of 5 years’ duration asks your opinion regarding pancreas transplantation. He is concerned that in spite of his best efforts, it is very likely that he will develop both microvascular and macrovascular complications.

Which of the following statements about pancreas transplantation is false?
A. Recipients of pancreas transplantation usually have normal insulin levels after successful transplantation
B. Pancreas transplantation can prevent or reduce nephropathy in diabetic patients with kidney transplants
C. The graft pancreas is usually placed in the right lower quadrant, with vascular anastomoses to the common iliac artery and common iliac vein or portal vein
D. Rejection is the leading cause of graft loss after transplantation
E. A major difficulty with islet cell transplantation is that more than one pancreas is required to provide enough islets for the recipient to become euglycemic

Key Concept/Objective: To understand the metabolic benefits of pancreas transplantation and the complications associated with this type of transplantation

Pancreas transplantation has been shown to prevent or reduce the nephropathy that often develops in kidney grafts in diabetic patients. The favored placement of the graft is the right lower quadrant, with vascular anastomoses to the common iliac artery and the common iliac vein or portal vein; in simultaneous pancreas and kidney transplantations, the preferred placement is the left lower quadrant. Rejection is the leading cause of graft loss; vascular thrombosis is the leading nonimmunologic cause. Glucose tolerance tests are usually normal or near normal for pancreas transplant recipients. However, insulin levels are much higher than normal in these patients. (Answer: A—Recipients of pancreas transplantation usually have normal insulin levels after successful transplantation)

48. Three months after liver transplantation for chronic hepatitis C infection, a 45-year-old man develops biochemical abnormalities suggestive of cholestatic hepatitis.

Which of the following evaluation strategies is most important for this patient at this time?
A. Cytomegalovirus (CMV) culture of the blood
B. Hepatitis C virus (HCV) RNA levels
C. Endoscopic retrograde cholangiopancreatography (ERCP)
D. Doppler ultrasonography to look for hepatic artery thrombosis
E. Liver biopsy

Key Concept/Objective: To understand the evaluation of liver dysfunction 1 month and longer after liver transplantation

It is important to use liver biopsy to determine the specific cause of allograft dysfunction that occurs more than 30 days after transplantation. Neither serum hepatic enzyme levels nor measures of viral load can be reliably used to determine the specific cause of allograft dysfunction occurring as cholestatic hepatitis. Because HCV hepatitis, CMV hepatitis, and transplant rejection differ histologically, liver biopsy is important. If the biopsy results suggest biliary tract disease, ERCP would be performed. Doppler studies would be performed if there were histoloic evidence of ischemia. (Answer: E—Liver biopsy)

49. A 38-year-old woman with long-standing insulin-dependent diabetes mellitus has complications of nephropathy, retinopathy, peripheral neuropathy, and mild gastroparesis. She has been missing more and more time from work as a nurse administrator. She asks you about being referred for simultaneous pancreas and kidney transplantation. Assuming she is a candidate, your patient asks about what she might expect if she undergoes the procedure.

Which of the following outcomes has been shown to occur following successful simultaneous pancreas and kidney transplantation?
A. Improvement in macrovascular disease
B. Improvement in gastroparesis
C. Improvement in retinopathy
D. Lowered use of immunosuppressive agents because of normoglycemia
E. Increased likelihood of returning to full-time work

Key Concept/Objective: To understand outcomes associated with simultaneous pancreas and kidney transplantation

Well-controlled, prospective studies assessing the long-term benefits of improved glucose control have yet to be carried out. The studies that have been reported reveal no improvement for patients with macroangiopathy, retinopathy, and gastroparesis. Glucose control does occur but does not diminish the need for immunosuppressive agents. Quality of life improves, including the capacity to return to full-time or part-time work. (Answer: E— Increased likelihood of returning to full-time work)

For more information, see Carithers RL Jr., Perkins JD: 4 Gastroenterology: XV Liver and Pancreas Transplantation. ACP Medicine Online (www.acpmedicine.com). Dale DC, Federman DD, Eds. WebMD Inc., New York, November 2003